Compare commits

...

4 Commits

Author SHA1 Message Date
e962b1a314
thank you, Mirko!
Some checks failed
Build latex and deploy / checkout (push) Failing after 13m20s
2024-01-09 23:03:24 +01:00
cf5a26ec93
lecture 20 2024-01-09 22:49:22 +01:00
e7a325bac8
Merge branch 'main' of https://git.abstractnonsen.se/josia-notes/w23-logic-3 2024-01-09 20:24:24 +01:00
62fcb59f94
tutorial 10 2024-01-09 20:23:57 +01:00
12 changed files with 358 additions and 34 deletions

View File

@ -12,6 +12,9 @@ If you find errors or want to improve something,
please send me a message:\\ please send me a message:\\
\texttt{lecturenotes@jrpie.de}. \texttt{lecturenotes@jrpie.de}.
Many thanks to \textsc{Mirko Bartsch} for providing notes on lectures
I could not attend!
This notes follow the way the material was presented in the lecture rather This notes follow the way the material was presented in the lecture rather
closely. Additions (e.g.~from exercise sheets) closely. Additions (e.g.~from exercise sheets)
and slight modifications have been marked with $\dagger$. and slight modifications have been marked with $\dagger$.

View File

@ -61,7 +61,9 @@ However the converse of this does not hold.
Then $\{x_0\}$ is dense in $X$, but $X$ is not second countable. Then $\{x_0\}$ is dense in $X$, but $X$ is not second countable.
\end{example} \end{example}
\begin{example}[Sorgenfrey line] \begin{example}[Sorgenfrey line]
\todo{Counterexamples in Topology} Consider $\R$ with the topology given by the basis $\{[a,b) : a,b \in \R\}$.
This is T3, but not second countable
and not metrizable.
\end{example} \end{example}
\begin{fact} \begin{fact}
@ -98,6 +100,7 @@ However the converse of this does not hold.
then $X$ is metrisable. then $X$ is metrisable.
\end{theorem} \end{theorem}
\begin{absolutelynopagebreak}
\begin{fact} \begin{fact}
If $X$ is a compact Hausdorff space, If $X$ is a compact Hausdorff space,
the following are equivalent: the following are equivalent:
@ -107,6 +110,7 @@ However the converse of this does not hold.
\item $X$ is second countable. \item $X$ is second countable.
\end{itemize} \end{itemize}
\end{fact} \end{fact}
\end{absolutelynopagebreak}
\subsection{Some facts about polish spaces} \subsection{Some facts about polish spaces}
@ -175,6 +179,7 @@ suffices to show that open balls in one metric are unions of open balls in the o
\end{proof} \end{proof}
\begin{definition}[Our favourite Polish spaces] \begin{definition}[Our favourite Polish spaces]
\leavevmode
\begin{itemize} \begin{itemize}
\item $2^{\omega}$ is called the \vocab{Cantor set}. \item $2^{\omega}$ is called the \vocab{Cantor set}.
(Consider $2$ with the discrete topology) (Consider $2$ with the discrete topology)
@ -185,7 +190,7 @@ suffices to show that open balls in one metric are unions of open balls in the o
\end{itemize} \end{itemize}
\end{definition} \end{definition}
\begin{proposition} \begin{proposition}
Let $X$ be a separable, metrisable topological space. Let $X$ be a separable, metrisable topological space\footnote{e.g.~Polish, but we don't need completeness.}.
Then $X$ topologically embeds into the Then $X$ topologically embeds into the
\vocab{Hilbert cube}, \vocab{Hilbert cube},
i.e. there is an injective $f: X \hookrightarrow [0,1]^{\omega}$ i.e. there is an injective $f: X \hookrightarrow [0,1]^{\omega}$
@ -227,7 +232,9 @@ suffices to show that open balls in one metric are unions of open balls in the o
$f^{-1}$ is continuous. $f^{-1}$ is continuous.
\end{claim} \end{claim}
\begin{subproof} \begin{subproof}
\todo{Exercise!} Consider $B_{\epsilon}(x_n) \subseteq X$ for some $n \in \N$, $\epsilon > 0$.
Then $f(U) = f(X) \cap [0,1]^{n} \times [0,\epsilon) \times [0,1]^{\omega}$
is open\footnote{as a subset of $f(X)$!}.
\end{subproof} \end{subproof}
\end{proof} \end{proof}
\begin{proposition} \begin{proposition}

View File

@ -13,7 +13,6 @@
\begin{proof} \begin{proof}
Let $C \subseteq X$ be closed. Let $C \subseteq X$ be closed.
Let $U_{\frac{1}{n}} \coloneqq \{x | d(x, C) < \frac{1}{n}\}$. Let $U_{\frac{1}{n}} \coloneqq \{x | d(x, C) < \frac{1}{n}\}$.
\todo{Exercise}
Clearly $C \subseteq \bigcap U_{\frac{1}{n}}$. Clearly $C \subseteq \bigcap U_{\frac{1}{n}}$.
Let $x \in \bigcap U_{\frac{1}{n}}$. Let $x \in \bigcap U_{\frac{1}{n}}$.
Then $\forall n .~ \exists x_n\in C.~d(x,x_n) < \frac{1}{n}$. Then $\forall n .~ \exists x_n\in C.~d(x,x_n) < \frac{1}{n}$.
@ -54,7 +53,7 @@
then there exists a complete metric on $Y$. then there exists a complete metric on $Y$.
\end{claim} \end{claim}
\begin{refproof}{psubspacegdelta:c1} \begin{refproof}{psubspacegdelta:c1}
Let $Y = U$be open in $X$. Let $Y = U$ be open in $X$.
Consider the map Consider the map
\begin{IEEEeqnarray*}{rCl} \begin{IEEEeqnarray*}{rCl}
f_U\colon U &\longrightarrow & f_U\colon U &\longrightarrow &

View File

@ -1,6 +1,6 @@
\lecture{03}{2023-10-17}{Embedding of the cantor space into polish spaces} \lecture{03}{2023-10-17}{Embedding of the cantor space into polish spaces}
% ? \subsection{Trees} TODO \subsection{Trees}
\begin{notation} \begin{notation}
@ -255,9 +255,7 @@
[To be continued]
\phantom\qedhere \phantom\qedhere
\end{refproof} \end{refproof}

View File

@ -1,6 +1,7 @@
\lecture{04}{2023-10-20}{} \lecture{04}{2023-10-20}{}
\begin{remark} \begin{remark}
Some of $F_s$ might be empty. Some of the $F_s$ might be empty.
\end{remark} \end{remark}
\begin{refproof}{thm:bairetopolish} \begin{refproof}{thm:bairetopolish}
@ -29,7 +30,6 @@
\begin{refproof}{thm:bairetopolish:c1} \begin{refproof}{thm:bairetopolish:c1}
Let $x_n$ be a series in $D$ Let $x_n$ be a series in $D$
converging to $x$ in $\cN$. converging to $x$ in $\cN$.
Then $x \in \cN$.
\begin{claim} \begin{claim}
$(f(x_n))$ is Cauchy. $(f(x_n))$ is Cauchy.
\end{claim} \end{claim}
@ -40,21 +40,19 @@
$x_m\defon{N} = x\defon{N}$. $x_m\defon{N} = x\defon{N}$.
Then for all $m, n \ge M$, Then for all $m, n \ge M$,
we have that $f(x_m), f(x_n) \in F_{x\defon{N}}$. we have that $f(x_m), f(x_n) \in F_{x\defon{N}}$.
So $d(f(x_m), f(x_n)) < \epsilon$ So $d(f(x_m), f(x_n)) < \epsilon$, i.e.~$(f(x_n))$ is Cauchy.
we have that $(f(x_n))$ is Cauchy.
Since $(X,d)$ is complete,
there exists $y = \lim_n f(x_n)$.
Since for all $m \ge M$, $f(x_m) \in F_{x\defon{N}}$,
we get that $y \in \overline{F_{x\defon{N}}}$.
Note that for $N' > N$ by the same argument
we get $y \in \overline{F_{x\defon{N'}}}$.
Hence
\[y \in \bigcap_{n} \overline{F_{x\defon{n}}} = \bigcap_{n} F_{x\defon{n}},\]
i.e.~$y \in D$ and $y = f(x)$.
\end{subproof} \end{subproof}
Since $(X,d)$ is complete,
there exists $y = \lim_n f(x_n)$.
Since for all $m \ge M$, $f(x_m) \in F_{x\defon{N}}$,
we get that $y \in \overline{F_{x\defon{N}}}$.
Note that for $N' > N$ by the same argument
we get $y \in \overline{F_{x\defon{N'}}}$.
Hence
\[y \in \bigcap_{n} \overline{F_{x\defon{n}}} = \bigcap_{n} F_{x\defon{n}},\]
i.e.~$y \in D$ and $y = f(x)$.
\end{refproof} \end{refproof}
We extend $f$ to $g\colon\cN \to X$ We extend $f$ to $g\colon\cN \to X$
@ -70,7 +68,7 @@
(i.e.~$r = \id$ on $D$ and $r$ is a continuous surjection). (i.e.~$r = \id$ on $D$ and $r$ is a continuous surjection).
Then $g \coloneqq f \circ r$. Then $g \coloneqq f \circ r$.
To construct $r$, we will define by induction To construct $r$, we will define
$\phi\colon \N^{<\N} \to S$ by induction on the length $\phi\colon \N^{<\N} \to S$ by induction on the length
such that such that
\begin{itemize} \begin{itemize}

View File

@ -15,6 +15,7 @@ sketch of \yaref{thm:l16:3} in the case of $|Z| = 1$.
Let $(X,T)$ be a distal flow. Let $(X,T)$ be a distal flow.
Then $G \coloneqq E(X,T)$ is a group. Then $G \coloneqq E(X,T)$ is a group.
\begin{definition} \begin{definition}
\label{def:F}
For $x, x' \in X$ define For $x, x' \in X$ define
\[ \[
F(x,x') \coloneqq \inf \{d(gx, gx') : g \in G\}. F(x,x') \coloneqq \inf \{d(gx, gx') : g \in G\}.

View File

@ -274,6 +274,7 @@ More generally we can show:
$(X'_{\xi+1}, T)$ is a factor of $(X_{\xi+1}, T)$. $(X'_{\xi+1}, T)$ is a factor of $(X_{\xi+1}, T)$.
\end{proof} \end{proof}
\begin{example}[{\cite[p. 513]{Furstenberg}}] \begin{example}[{\cite[p. 513]{Furstenberg}}]
\label{ex:19:inftorus}
Let $X$ be the infinite torus Let $X$ be the infinite torus
\[ \[
X \coloneqq \{(\xi_1, \xi_2, \ldots) : \xi_i \in \C, |\xi_i| = 1\}. X \coloneqq \{(\xi_1, \xi_2, \ldots) : \xi_i \in \C, |\xi_i| = 1\}.

196
inputs/lecture_20.tex Normal file
View File

@ -0,0 +1,196 @@
\lecture{20}{2024-01-09}{The infinite Torus}
\begin{example}
\footnote{This is the same as \yaref{ex:19:inftorus},
but with new notation.}
Let $X = (S^1)^{\N}$\footnote{We identify $S^1$ and $\faktor{\R}{\Z}$.}
and consider $\left(X, \Z \right)$
where the action is generated by
\[
\tau\colon (x_1,x_2,x_3,\ldots) \mapsto(x_1 + \alpha, x_1 + x_2, x_2 + x_3, \ldots)
\]
for some irrational $\alpha$.
\end{example}
\begin{remark}+
Note that we can identify $S^1$ with a subset of $\C$ (and use multiplication)
or with $\faktor{\R}{\Z}$ (and use addition).
In the lecture both notations were used.% to make things extra confusing.
Here I'll try to only use multiplicative notation.
\end{remark}
We will be studying projections to the first $d$ coordinates,
i.e.
\[
\tau_d \colon (x_1,\ldots,x_d) \mapsto (e^{\i \alpha} x_1, x_1x_2, \ldots, x_{d-1}x_d).
\]
$\tau_d$ is called the \vocab{$d$-skew shift}.
For $d = 1$ we get the circle rotation $x \mapsto e^{\i \alpha} x$.
\begin{fact}
\label{fact:tau1minimal}
The circle rotation $x \mapsto e^{\i \alpha} x$ is minimal.
In fact, every subgroup of $S^1$ is either dense in $S^1$
or it is of the form
\[
H_m \coloneqq \{x \in S^1 : x^m = 0\}
\]
for some $m \in \Z$.
\end{fact}
\todo{Homework!}
We will show that $\tau_d$ is minimal for all $d$,
i.e.~every orbit is dense.
From this it will follow that $\tau$ is minimal.
Let $\pi_n\colon X \to (S^1)^n$ be the projection to the first $n$
coordinates.
\begin{lemma}
Let $x,x' \in X$ with $\pi_n(x) = \pi_n(x')$
for some $n$.
Then there is a sequence of points $x_k$ with
\[\pi_{n-1}(x_k) = \pi_{n-1}(x) = \pi_{n-1}(x')\]
for all $k$
and
\[
F(x_k, x) \xrightarrow{k \to \infty} 0,
F(x_k, x') \xrightarrow{k \to \infty} 0,
\]
where $F$ is as in \yaref{def:F},
i.e.~$F(a,b) = \inf_{n \in \Z} d(\tau^n a, \tau^n b)$,
where $d$ is the metric on $X$,
$d((x_i), (y_i)) = \max_n \frac{1}{2^n} | x_n - y_n|$.% TODO use multiplicative notation
\end{lemma}
\begin{proof}
Let
\begin{IEEEeqnarray*}{rCl}
x &=& (\alpha^0_1, \alpha^0_2, \ldots, \alpha^0_{n-1}, \alpha_n, \alpha_{n+1}, \alpha_{n+2},\ldots)\\
x' &=& (\alpha^0_1, \alpha^0_2, \ldots, \alpha^0_{n-1}, \alpha_n, \alpha'_{n+1}, \alpha'_{n+2},\ldots).\\
\end{IEEEeqnarray*}
We will choose $x_k$ of the form
\[
(\alpha^0_1, \alpha^0_2, \ldots, \alpha^0_{n-1} \alpha_n e^{\i \beta_k}, \alpha_{n+1}, \alpha_{n+2}, \ldots),
\]
where $\beta_k$ is such that $\frac{\beta_k}{\pi}$ is irrational
and $|\beta_k| < 2^{-k}$.
Fix a sequence of such $\beta_k$.
Then
\[d(x_k,x) = 2^{-n} |e^{\i \beta_k} - 1| < 2^{-n-k} \xrightarrow{k\to \infty} 0.\]
In particular $F(x_k, x) \to 0$.
We want to show that $F(x_k, x') < 2^{-n-k}$.
For $u, u' \in X$,
$u = (\xi_n)_{n \in \N}$,
$u' = (\xi'_n)_{n \in \N}$,
let $\frac{u}{u'} = (\frac{\xi_n}{\xi'_n})_{n \in \N}$
($X$ is a group).
We are interested in $F(x_k, x') = \inf_m d(\tau^m x_k, \tau^m x')$,
but it is easier to consider the distance between
their quotient and $1$.
Consider
\[
w_k \coloneqq \frac{x_k}{x'} = (\underbrace{1,\ldots,1}_{n-1}, e^{\i \beta_k}, \overbrace{\frac{\alpha_{n+1}}{\alpha'_{n+1}}, \frac{\alpha_{n+2}}{\alpha'_{n+2}}, \ldots}^{\mathclap{\text{not interesting}}}).
\]
\begin{claim}
$F(x_k, x') = \inf_m d(\sigma^m(w_k), 1)$,
where $\sigma(\xi_1, \xi_2, \ldots) = (\xi_1, \xi_1\xi_2, \xi_2\xi_3, \ldots)$.
\end{claim}
\begin{subproof}
We have
\begin{IEEEeqnarray*}{rCl}
F(u,u') &=& \inf_m d(\tau^m u, \tau^m u')\\
&=& \inf_m d(\frac{\tau^m u}{\tau^m u'}, 1)\\
&=& \inf_m d(\sigma^m\left( \frac{u}{u'} \right), 1).
\end{IEEEeqnarray*}
\end{subproof}
Fix $k$. Let $w^\ast = (1,\ldots,1, e^{\i \beta_k}, 1, \ldots)$.
By minimality of $(X,T)$ for any $\epsilon >0$,
there exists $m \in \Z$ such that
$d(\sigma^m w_k, w^\ast) < \epsilon$.
% TODO Think about this
Then
\begin{IEEEeqnarray*}{rCl}
\inf_m d(\sigma^m w_k, 1) &\le & \inf_m d(\sigma^m w_k, w^\ast) + d(w^\ast, 1)\\
&\le & 2^{-n} | e^{\i \beta_k}- 1|\\
&<& 2^{-n-k}.
\end{IEEEeqnarray*}
\end{proof}
\begin{definition}
For every continuous $f\colon S^1 \to S^1$, the
\vocab{winding number} $[f] \in \Z$
is the unique integer such that $f$ is homotopic%
\footnote{$f\colon Y \to Z$ and $g\colon Y \to Z$ are homotopic
iff there is $H\colon Y \times [0,1] \to \Z$
continuous such that $H(\cdot ,0) = f$ and $H(\cdot ,1) = g$.}
to the map
$x \mapsto x^{n}$.
\end{definition}
\begin{remark}
Note that for
\begin{IEEEeqnarray*}{rCl}
\sigma\colon (S^1)^d &\longrightarrow & S^1 \\
(x_1,\ldots,x_d) &\longmapsto & x_d
\end{IEEEeqnarray*}
we have that $T = \tau_{d+1}$,
where
\begin{IEEEeqnarray*}{rCl}
T\colon (S^1)^d \times S^1 &\longrightarrow & (S^1)^d \times S^1 \\
(y, x_{d+1}) &\longmapsto & (\tau_d(y), \sigma(y) x_{d+1}).
\end{IEEEeqnarray*}
\end{remark}
\begin{theorem}
\label{thm:taudminimal:help}
For every $d$ if $\tau_d$\footnote{more formally $((S^1)^d, \langle \tau_d \rangle)$}
is minimal, then $\tau_{d+1}$ is minimal.
\end{theorem}
\begin{corollary}
$\tau_d$ is minimal for all $d$.
\end{corollary}
\begin{proof}
$\tau_1$ is minimal (\yaref{fact:tau1minimal}).
Apply \yaref{thm:taudminimal:help}.
\end{proof}
\begin{corollary}
Since all the $\tau_d$ are minimal,
$\tau$ is minimal.
\end{corollary}
\begin{proof}
This follows from the definition of the product topology,
since for a basic open set $U = U_1 \times \ldots \times U_d \times (S^1)^{\infty}$
it suffices to analyze the first $d$ coordinates.
\end{proof}
\begin{refproof}{thm:taudminimal:help}
Let $s \coloneqq \tau_d$ and $Y \coloneqq (S^1)^d$.
Consider
\begin{IEEEeqnarray*}{rCl}
\gamma\colon S^1 &\longrightarrow & Y \\
x &\longmapsto & (x,x,\ldots,x)
\end{IEEEeqnarray*}
\begin{enumerate}[(a)]
\item $\gamma$ and $s \circ \gamma$ are homotopic
via
\begin{IEEEeqnarray*}{rCl}
H\colon S^1 \times [0,1] &\longrightarrow & (S^1)^d \\
(x, t)&\longmapsto & (x e^{\i t \alpha}, x^{t+1}, x^{t+1}, x^{t+1},\ldots, x^{t+1})
\end{IEEEeqnarray*}
\item For all $m \in \Z \setminus \{0\}$, we have
$[x \mapsto \left(\sigma(\gamma(x))\right)^m] = m \neq 0$,
since $\sigma(\gamma(x)) = \sigma((x,\ldots,x)) = x$.
\end{enumerate}
[to be continued]
\phantom\qedhere
\end{refproof}

View File

@ -13,9 +13,9 @@
\begin{fact} \begin{fact}
\begin{itemize} \begin{itemize}
\item Let $X$ be a topological space. \item Let $X$ be a topological space.
Then $X$ 2nd countable $\implies$ X separable. Then $X$ \nth{2} countable $\implies$ X separable.
\item If $X$ is a metric space and separable, \item If $X$ is a metric space and separable,
then $X$ is 2nd countable. then $X$ is \nth{2} countable.
\end{itemize} \end{itemize}
\end{fact} \end{fact}
\begin{proof} \begin{proof}
@ -32,18 +32,18 @@
\begin{fact} \begin{fact}
Let $X$ be a metric space. Let $X$ be a metric space.
If $X$ is Lindelöf, If $X$ is Lindelöf,
then it is 2nd countable. then it is \nth{2} countable.
\end{fact} \end{fact}
\begin{proof} \begin{proof}
For all $q \in \Q$ For all $q \in \Q$
Consider the cover $B_q(x), x \in X$ consider the cover $B_q(x), x \in X$
and choose a countable subcover. and choose a countable subcover.
The union of these subcovers is The union of these subcovers is
a countable base. a countable base.
\end{proof} \end{proof}
\begin{fact} \begin{fact}
Let $X$ be a topological space. Let $X$ be a topological space.
If $X$ is 2nd countable, If $X$ is \nth{2} countable,
then it is Lindelöff. then it is Lindelöff.
\end{fact} \end{fact}
\begin{proof} \begin{proof}
@ -60,7 +60,7 @@
\end{proof} \end{proof}
\begin{remark} \begin{remark}
For metric spaces the notions For metric spaces the notions
of being 2nd countable, separable of being \nth{2} countable, separable
and Lindelöf coincide. and Lindelöf coincide.
In arbitrary topological spaces, In arbitrary topological spaces,

113
inputs/tutorial_11.tex Normal file
View File

@ -0,0 +1,113 @@
\tutorial{11}{2024-01-09}{}
An equivalent definition of subflows can be given as follows:
\begin{definition}
Let $(X,T)$ be a flow with action $\alpha_x$.
Let $Y \subseteq X$ be a compact subspace of $X$.
If $Y$ is invariant under $\alpha_x$, we say that
$(Y,T)$ (with action $\alpha_x\defon{T \times Y}$
is a subflow of $(X,T)$.
\end{definition}
\begin{example}[Flows with a non-closed orbit]
\begin{enumerate}[1.]
\item Consider $(S^1, \Z)$
with action given by $1 \cdot x = x + c$ for
a fixed $c \in \R\setminus\Q$.\footnote{We identify $S^1$ and $\faktor{\R}{\Z}$.}
Then the orbit of $0$, $\{nc : n \in \Z\}$ is dense but consists only of irrationals
(except $0$),
so it is not closed.
\item Consider $(S^1, \Q)$ with action $qx \coloneqq x + q$.
The orbit of $0$, $\faktor{\Q}{\Z} \subseteq S^1$,
is dense but not closed.
$(S^1,\Q)$ is minimal.
\end{enumerate}
\end{example}
\begin{example}[\vocab{Left Bernoulli shift}]
Consider $(\{0,1\}^{\Z}, T)$,
where $T = \Z$ and the action is given by
\begin{IEEEeqnarray*}{rCl}
\Z \times \{0,1\}^{\Z}&\longrightarrow & \Z \\
(m, (x_n)_{n \in \Z})&\longmapsto & (x_{n+m})_{n \in \Z}.
\end{IEEEeqnarray*}
The orbit of $z \coloneqq (0)_{n \in \Z}$ consist of only on point.
In particular it is closed.
Let $x \coloneqq ( [n = 0])_{n \in \Z}$.
Then $Tx = \{([n = m])_{n \in \Z} | m \in \Z\}$.
Clearly $z \not\in Tx$.
\begin{claim}
$z \in \overline{Tx}$
\end{claim}
\begin{proof}
Consider a basic open $z \in U_I = \{y : y_i = 0, i \in I\}$
where $I \subseteq \Z$ is finite.
Then $U_I \cap Tx \neq \emptyset$
as we can shift the $1$ out of $I$,
i.e.~$(\max I + 1) x \in U_I$.
\end{proof}
\end{example}
Flows are always on non-empty spaces $X$.
\begin{fact}
Consider a flow $(X,T)$.
The following are equivalent:
\begin{enumerate}[(i)]
\item Every $T$-orbit is dense.
\item There is no proper subflow,
\end{enumerate}
If these conditions hold, the flow is called \vocab{minimal}.
\end{fact}
\begin{proof}
(i) $\implies$ (ii):
Let $(Y,T)$ be a subflow of $(X,T)$.
take $y \in Y$. Then $Ty$ is dense in mKX.
But $Ty \subseteq Y$, so $Y$ is dense in $X$.
Since $Y$ is closed, we get $Y = X$.
(ii) $\implies$ (i):
Take $x \in X$. Consider $Tx$.
It suffices to show that $\overline{Tx}$ is a subflow.
Clearly $\overline{Tx}$ is closed,
so it suffices to show that it is $T$-invariant.
Let $y \in \overline{Tx}$ and $t \in T$.
Take $ty \in U \overset{\text{open}}{\subseteq} X$.
Since $t^{-1}$ acts as a homeomorphism
we have $y \in t^{-1} U \overset{\text{open}}{\subseteq} X$.
We find some $t'x \in t^{-1}U$ since $y \in \overline{Tx}$.
So $tt'x \in Tx \cap U$.
\end{proof}
\begin{fact}
Every flow $(X,T)$ contains a minimal subflow.
\end{fact}
\begin{proof}
We use Zorn's lemma:
Let $S$ be the set of all subflows of $(X,T)$
ordered by $Y \le Y' :\iff Y \supseteq Y'$.
We need to show that for a chain $\langle Y_i : i \in I \rangle$,
there exists a lower bound.
Consider $\bigcap_{i \in I} Y_i$. This a subflow:
\begin{itemize}
\item It is closed as it is an intersection of closed sets.
\item It is $T$-invariant, since each of the $Y_i$ is.
\item It is non-empty by \yaref{tut10fact}.
\end{itemize}
\end{proof}
\begin{fact}
\label{tut10fact}
Let $X$ be a topological space.
Then $X$ is compact iff every family of closed sets with
FIP\footnote{finite intersection property, i.e.~the intersection of every finite sub-family is non-empty}
has non-empty intersection.
\end{fact}
\begin{proof}
Note that families of
closed sets correspond to families of open sets by taking complements.
A family of open sets is a cover iff the corresponding family
has empty intersection,
and is admits a finite subcover iff the corresponding family
has the FIP.
\end{proof}

View File

@ -50,3 +50,13 @@
\RequirePackage{mkessler-mathfixes} % Load this last since it renews behaviour \RequirePackage{mkessler-mathfixes} % Load this last since it renews behaviour
\DeclareMathOperator{\inter}{int} % interior \DeclareMathOperator{\inter}{int} % interior
\newcommand{\defon}[1]{|_{#1}} % TODO \newcommand{\defon}[1]{|_{#1}} % TODO
\RequirePackage[super]{nth}
% TODO MOVE
% https://tex.stackexchange.com/a/94702
\newenvironment{absolutelynopagebreak}
{\par\nobreak\vfil\penalty0\vfilneg
\vtop\bgroup}
{\par\xdef\tpd{\the\prevdepth}\egroup
\prevdepth=\tpd}

View File

@ -44,10 +44,7 @@
\input{inputs/lecture_17} \input{inputs/lecture_17}
\input{inputs/lecture_18} \input{inputs/lecture_18}
\input{inputs/lecture_19} \input{inputs/lecture_19}
\input{inputs/lecture_20}
\cleardoublepage \cleardoublepage
@ -64,6 +61,7 @@
\input{inputs/tutorial_08} \input{inputs/tutorial_08}
\input{inputs/tutorial_09} \input{inputs/tutorial_09}
\input{inputs/tutorial_10} \input{inputs/tutorial_10}
\input{inputs/tutorial_11}
\section{Facts} \section{Facts}
\input{inputs/facts} \input{inputs/facts}